Difference between revisions of "1999 AHSME Problems/Problem 20"

(Solution)
m
 
(5 intermediate revisions by 2 users not shown)
Line 1: Line 1:
 
== Problem ==
 
== Problem ==
The sequence <math>a_{1},a_{2},a_{3},\ldots</math> statisfies <math>a_{1} = 19,a_{9} = 99</math>, and, for all <math>n\geq 3</math>, <math>a_{n}</math> is the arithmetic mean of the first <math>n - 1</math> terms.  Find <math>a_2</math>.
+
The sequence <math>a_{1},a_{2},a_{3},\ldots</math> satisfies <math>a_{1} = 19,a_{9} = 99</math>, and, for all <math>n\geq 3</math>, <math>a_{n}</math> is the arithmetic mean of the first <math>n - 1</math> terms.  Find <math>a_2</math>.
  
== Solution ==
+
<math>\textrm{(A)} \ 29 \qquad \textrm{(B)} \ 59 \qquad \textrm{(C)} \ 79 \qquad \textrm{(D)} \ 99 \qquad \textrm{(E)} \ 179</math>
 +
 
 +
== Solution 1==
  
 
Let <math>m</math> be the arithmetic mean of <math>a_1</math> and <math>a_2</math>. We can then write <math>a_1=m-x</math> and <math>a_2=m+x</math> for some <math>x</math>.
 
Let <math>m</math> be the arithmetic mean of <math>a_1</math> and <math>a_2</math>. We can then write <math>a_1=m-x</math> and <math>a_2=m+x</math> for some <math>x</math>.
Line 13: Line 15:
  
 
It follows that <math>m=a_9=99</math>. From <math>19=a_1=m-x</math> we get that <math>x=80</math>. And thus <math>a_2 = m+x = \boxed{(E)  179}</math>.
 
It follows that <math>m=a_9=99</math>. From <math>19=a_1=m-x</math> we get that <math>x=80</math>. And thus <math>a_2 = m+x = \boxed{(E)  179}</math>.
 +
 +
== Solution 2==
 +
Let <math>a_1=a</math> and <math>a_2=b</math>. Then, <math>a_3=\frac{a+b}{2}</math>, <math>a_4=\frac{a+b+\frac{a+b}{2}}{3}=\frac{a+b}{2},</math> and so on.
 +
 +
Since <math>a_3=a_4</math>, <math>a_n=a_3</math> for all <math>n\geq3.</math>
 +
 +
Hence, <math>a_9=\frac{a_1+a_2}{2}=\frac{a+b}{2}=99, a+b=198.</math> We also know that <math>a_1=a=19.</math>
 +
 +
Subtracting <math>a_1</math> from <math>198,</math> we get <math>b=a_2=\boxed{(E)  179}.</math>
 +
 +
~Benedict T (countmath1)
  
 
== See also ==
 
== See also ==

Latest revision as of 14:00, 21 March 2023

Problem

The sequence $a_{1},a_{2},a_{3},\ldots$ satisfies $a_{1} = 19,a_{9} = 99$, and, for all $n\geq 3$, $a_{n}$ is the arithmetic mean of the first $n - 1$ terms. Find $a_2$.

$\textrm{(A)} \ 29 \qquad \textrm{(B)} \ 59 \qquad \textrm{(C)} \ 79 \qquad \textrm{(D)} \ 99 \qquad \textrm{(E)} \ 179$

Solution 1

Let $m$ be the arithmetic mean of $a_1$ and $a_2$. We can then write $a_1=m-x$ and $a_2=m+x$ for some $x$.

By definition, $a_3=m$.

Next, $a_4$ is the mean of $m-x$, $m+x$ and $m$, which is again $m$.

Realizing this, one can easily prove by induction that $\forall n\geq 3;~ a_n=m$.

It follows that $m=a_9=99$. From $19=a_1=m-x$ we get that $x=80$. And thus $a_2 = m+x = \boxed{(E)  179}$.

Solution 2

Let $a_1=a$ and $a_2=b$. Then, $a_3=\frac{a+b}{2}$, $a_4=\frac{a+b+\frac{a+b}{2}}{3}=\frac{a+b}{2},$ and so on.

Since $a_3=a_4$, $a_n=a_3$ for all $n\geq3.$

Hence, $a_9=\frac{a_1+a_2}{2}=\frac{a+b}{2}=99, a+b=198.$ We also know that $a_1=a=19.$

Subtracting $a_1$ from $198,$ we get $b=a_2=\boxed{(E)  179}.$

~Benedict T (countmath1)

See also

1999 AHSME (ProblemsAnswer KeyResources)
Preceded by
Problem 19
Followed by
Problem 21
1 2 3 4 5 6 7 8 9 10 11 12 13 14 15 16 17 18 19 20 21 22 23 24 25 26 27 28 29 30
All AHSME Problems and Solutions

The problems on this page are copyrighted by the Mathematical Association of America's American Mathematics Competitions. AMC logo.png